EE Gate 2021

You might also like

Download as pdf or txt
Download as pdf or txt
You are on page 1of 53

|EE-2021|

GENERAL APTITUDE

1. Let X be a continuous random variable denoting the temperature measured. The range of temperature is
[0, 100] degree Celsius and let the probability density function of X be f(x) = 0.01 for 0  X  100.
The mean of X is _______.
(A) 25.0 (B) 5.0 (C) 50.0 (D) 2.5
Key: (C)
Sol: E ( X ) = Mean of X
100
=  x.f ( x ) dx
0
100
100
 x2  0.01 1
=  x.( 0.01) dx = 0.01  = (100 ) =  100  100 = 50
2

0  2 0 2 200

2.

10 cm

10 cm

In the figure shown above, each inside square is formed by joining the midpoints of the sides of the next
larger square. The area of the smallest square (shaded) as shown, in cm2 is:
(A) 6.25 (B) 3.125 (C) 1.5625 (D) 12.50
Key: (B)
Sol: Using Pythagoras theorem, in each square, we can find side length.
5
Side of smallest square becomes =
8
5 5 25
Area of smallest square =  = = 3.125
8 8 8

© All rights reserved by Thinkcell Learning Solutions Pvt. Ltd. No part of this booklet may be reproduced or utilized in any form without the written permission.
|EE-2021|

3. The important of sleep is often overlooked by students when they are preparing for exams. Research has
consistently shown that sleep deprivation greatly reduces the ability to recall the material learnt. Hence,
cutting down on sleep to study longer hours can be counterproductive.
Which one of the following statements is the CORRECT inference from the above passage?
(A) To do well in exam, adequate sleep must be part of the preparation
(B) Students are efficient and are not wrong in thinking that sleep is a waste of time
(C) If a student is extremely well prepared for an exam, he needs little or no sleep
(D) Sleeping well alone is enough to prepare for an exam. Studying has lesser benefit
Key: (A)

4.
70
60
60
50 50
50
40
30
20
10 10
5
3
0
Year 1 Year 2 Year 3
Pass Fail

The number of students passing or failing in an exam for a particular subject is presented in the bar chart
above. Students who pass the exam cannot appear for the exam again. Students who fail the exam in the
first attempt must appear for the exam in the following year. Students always pass the exam in their
second attempt.
The number of students who took the exam for the first time in the year 2 and the year 3 respectively,
are________.
(A) 65 and 53 (B) 55 and 53 (C) 60 and 50 (D) 55 and 48
Key: (D)
Sol: From the bar graph
In year 2: Total number of students who took the exam = 60 + 5 = 65
But who took the exam for 1st time = 65 – 10 = 55.
In year 3: Total number of students who took the exam = 50 + 3 = 53
But who took the exam for 1st exam = 53 –5 = 48
Hence option (D) is correct.

© All rights reserved by Thinkcell Learning Solutions Pvt. Ltd. No part of this booklet may be reproduced or utilized in any form without the written permission.
|EE-2021|

5. Which one of the following numbers is exactly divisible by 1113 + 1 ? ( )


(A) 1152 − 1 (B) 1139 − 1 (C) 1133 + 1 (D) 1126 + 1
Key: (A)

1152 = (1126 ) − 12
2
Sol:
= (1126 + 1)(1126 − 1)

= (1126 + 1) (1113 ) − (12 ) 


2

 
= (1126 + 1) (1113 ) − 1 (1113 + 1)

1113 + 1 is divisor of 1152 −1.

6. Seven cars P, Q, R, S, T, U and V are parked in a row not necessarily in that order. The cars T and U
should be parked next to each other. The cars S and V also should be parked next to each other, whereas
P and Q cannot be parked next to each other. Q and S must be parked next to each other. R isparked to
the immediate right of V. T is parked to the left of U.
Based on the above statements, the only INCORRECT option given below is:
(A) There are two cars parked in between Q and V
(B) Q and R are not parked together
(C) V is the only car parked in between S and R
(D) Car P is parked at the extreme end
Key: (A)
Sol: According to the given information, we can arrange:
Q S V R T U P

Statements: (B), (C), (D) are true and


Statements (A) is incorrect.
( There is only one car between Q and V)

7.

A transparent square sheet show above is folded along the dotted line. The folded sheet will look like

© All rights reserved by Thinkcell Learning Solutions Pvt. Ltd. No part of this booklet may be reproduced or utilized in any form without the written permission.
|EE-2021|

(A) (B)

(C) (D)

Key: (C)
Sol: If the square sheet is folded along the dotted line, then the folded sheet will look like as follows.

8. The people _____________ were at the demonstration were from all sections of society.
(A) whose (B) which (C) whom (D) who
Key: (D)

9. Oasis is to sand as island is to ___________.


Which one of the following options maintains a similar logical relation in the above sentence?
(A) Water (B) Mountain (C) Land (D) Stone
Key: (A)

10. For a regular polygon having 10 sides, the interior angle between the sides of the polygon, in degrees, is:
(A) 396 (B) 144 (C) 216 (D) 324
Key: (B)
Sol: The formula for calculating the sum of interior angles in a regular polygon is (n–2) × 180°.
Here n = 10
 The sum of interior angles of a decagon = (10 –2) × 180 = 8 × 180°.
8  180
 Each interior angle = = 144.
10

© All rights reserved by Thinkcell Learning Solutions Pvt. Ltd. No part of this booklet may be reproduced or utilized in any form without the written permission.
|EE-2021|

ELECTRICAL ENGINEERING

1. In a single-phase transformer, the total iron loss is 2500 W at nominal voltage of 440V and frequency
50Hz. The total iron loss is 850W at 220V and 25 Hz. Then, at nominal voltage and frequency, the
hysteresis loss and eddy current loss respectively are
(A) 900 W and 1600 W (B) 250 W and 600 W
(C) 1600 W and 900 W (D) 600 W and 250 W
Key: (A)
Sol: Given:
W1 = 2500W
V1 = 440V
f1 = 50Hz
W2 = 850W
V2 = 220V
f 2 = 25Hz
Iron loss, Wi = Wh + Wf …(A)
Hysteresis loss, Wh = Af …(i)

Wf = Bf 2 …(ii)
Substitute (i) and (ii) in (A) equation, we get
Wi = Af + Bf 2
Wi
= A + Bf …(B)
f
Substitute, f1 = 50Hz in (B)
2500
= A + 50B
50
50 = A + 50B …(1)
Substitute f2 = 25Hz in (B)
850
= A + 25B
25
34 = A + 25B …(2)
Solve (1) and (2) equations, we get
A = 18, B = 0.64

© All rights reserved by Thinkcell Learning Solutions Pvt. Ltd. No part of this booklet may be reproduced or utilized in any form without the written permission.
|EE-2021|

Substitute A and B values in equation (i) and (ii) to find Wh and We .


Wh = A  f = 18  50 = 900W
We = Fyf = 0.64  502 = 1600W
V2 V1 220 440
=  =
f2 f1 25 50
So, Bm2 = Bm1

2. A signal generator having a source resistance of 50  is set to generate a 1 kHz sinewave. Open circuit
terminal voltage is 10V peak-to-peak. Connecting a capacitor across the terminals reduces the voltage to
8V peak-to-peak. The value of this capacitor is ___________ F. (Round off to 2 decimal places).
Key: (2.38)
Sol: As per the given information, before connecting the capacitor the circuit looks like

50

5sin ( 2103 t ) ~

If peak amplitude is 5 then peak to peak amplitude is 10.


After connecting capacitor at load, the circuit should look like

50 +

5sin ( 2103 t ) ~ C Vo ( t ) = 4sin ( 210 t + )

If peak amplitude is 4 then peak to peak amplitude 8.


Here out interest is only amplitude not phase

R +

4
50 ~ C

© All rights reserved by Thinkcell Learning Solutions Pvt. Ltd. No part of this booklet may be reproduced or utilized in any form without the written permission.
|EE-2021|

1 jC 1
V0 ( j) = Vi ( j) = 50
R + 1 jC 1 + jRC
5
 V0 ( j) =
1 + ( RC )
2

5
4=
1 + ( RC )
2

2
5
 1 + ( RC ) = 
2

4
(1.25) −1 (1.25) −1
2 2
0.5625
C= = = = 2.38F
R ( 2 10 )  ( 50 )
3 2 9.86  1010
2 2 2

3. A 100 Hz square wave, switching between 0V and 5V, is applied to a CR high-pass filter circuit as
shown. The output voltage waveform across the resistor is 6.2V peak-to-peak. If the resistance R is
820  , then the value C is ___________ F. (Rounded off to 2 decimal places).

Input R Output

Key: (12.43)
Sol: Given,
C
Vi + +
+ −
VC
Input, Vi R Output, Vo
5V
− − Vo,P − P ( peak to peak )
0 T2 T 3T 2 t

From KVL, Vi − VC − V0 = 0  V0 = Vi − VC
(Charging)
T
For 0  t  , VC ( 0 ) = VC,min ( assumed )
2

© All rights reserved by Thinkcell Learning Solutions Pvt. Ltd. No part of this booklet may be reproduced or utilized in any form without the written permission.
|EE-2021|

VC (  ) = 5V ( input available is maximum 5V capacitor can change maximum upto 5 only)

As VC ( t ) = V ( ) + V ( 0) − V ( ) et 

VC ( t ) = 5 + ( VC,min − 5) e− t 

, VC = VC,max  VC,max = 5 + ( VC,min − 5 ) e − T 2 


T
At t = …(1)
2
(Discharging)
T VC ( 0 ) = VC,max ,
For  t  T,
2 V1 (  ) = 0
( t −T 2) ( t −T 2)
VC = 0 + ( VC,max − 0 ) e
− −
 
= VC,max e 

At t = T, VC = VC,min  VC,min = VC,max e− T 2  …(ii)

From above analysis the corresponding waveforms are,


Vi

5V

t
T2 T 3T 2
VC
VC,max
VC,min
t

Vo
= Vi − VC 5 − VC,min

−VC,max

Given, Vo,P −P = 6.2V

 (5 − VC,min ) − ( −VC,max ) = 6.2

 VC,max − VC,min = 1.2V …(iii)

1
Solving (i), (ii) and (iii),  = 0.0102 using T = sec
100
 RC = 0.0102  C = 12.43 F

© All rights reserved by Thinkcell Learning Solutions Pvt. Ltd. No part of this booklet may be reproduced or utilized in any form without the written permission.
|EE-2021|

4. Let f(t) be an even function, i.e., f ( − t ) = f ( t ) for all t. Let the Fourier transform of f(t) be defined as

dF ( )
F ( ) =  f (t)e
− jt
dt. Suppose = −F ( ) for all , and F ( 0 ) = 1. Then
−
d

(A) f ( 0 )  1 (B) f ( 0 ) = 0 (C) f ( 0 )  1 (D) f ( 0 ) = 1

Key: (C)
Sol: It is given that f ( − t ) = f ( t ) = f ( t ) , t
dF ( )
= −F ( ) , 
d
F(0) = 1

dF ( )
= −d
F ( )

−2
 nF ( ) = +C
2
2
− +C
 F ( ) = e 2

 F ( 0 ) = e0+ C
 1 = eC
C=0
2

So F ( ) = e 2

We know

 −
2

e− at 
2
e 4a
a
 t 2
1 − 2 −
If a = then e 2  2 e 2
2
2
− 1 − t2 2
e 2
 e
2

1 − t22
So, f ( t ) = e
2
1
f ( 0) = so f ( 0 )  1
2

© All rights reserved by Thinkcell Learning Solutions Pvt. Ltd. No part of this booklet may be reproduced or utilized in any form without the written permission.
|EE-2021|

5. Consider the table given:


Constructional feature Machine type Mitigation
(P) Damper bars (S) Induction motor (X) Hunting
(Q) Skewed rotor slots (T) Transformer (Y) Magnetic locking
(R) Compensating winding (U) Synchronous machine (Z) Armature reaction
(V) DC machine
The correct combination that relates the constructional feature, machine type and mitigation is
(A) P-V-X, Q-U-Z, R-T-Y (B) P-T-Y, Q-V-Z, R-S-X
(C) P-U-X, Q-V-Y, R-T-Z (D) P-U-X, Q-S-Y, R-V-Z
Key: (D)
Sol: (i) Synchronous machines are not self-starting machines. These machines are made self-starting by
providing a special winding in the rotor poles known as damper winding. When there is change in
load, excitation or change in other conditions of the systems rotor of the synchronous motor will
oscillate to and fro about an equilibrium position. These oscillationsbecomes more violent and
resulting in loss of synchronism of the motor and comes to halt.
(ii) If the rotor and the stator conductors are parallel to each other, there is a strong possibility of the
magnetic locking between the rotor and stator. Therefore, the rotor slots are skewed.
(iii) In order to neutralize the cross-magnetizing effect of armature reaction, a compensating winding is
used in DC machines. The compensating windings consist of a series coils embedded in slots in the
pole faces. These coils are connected in series with the armature.
Hence, PUX – QSY – RVZ’

6. Inductance is measured by
(A) Schering bridge (B) Kelvin bridge
(C) Wien bridge (D) Maxwell bridge
Key: (D)
Sol: For inductance measurement Maxwell’s bridge is used.
• Schering bridge is used for measurement of capacitance.
• Wien bridge is used for measurement of frequency.
• Kelvin bridge is used for measurement of resistance.

7. Suppose IA ,IB and IC are a set of unbalanced current phasors in a three-phase system. The phase-B zero-
sequence current IB0 = 0.10 p.u. If phase-A current IA = 1.10 p.u and phase-C current
IC = (1120 + 0.1) p.u. then IB in p.uis
(A) 1 − 120 + 0.10 (B) 1 − 240 + 0.10
(C) 1.1240 − 0.10 (D) 1.1 − 120 + 0.10

© All rights reserved by Thinkcell Learning Solutions Pvt. Ltd. No part of this booklet may be reproduced or utilized in any form without the written permission.
|EE-2021|

Key: (A)
Sol: Given, I B0 = 0.10 pu
I A = 1.10 pu
IC = (1120 + 0.1) pu

 IA  1 1 1   I A0 
We know,  IB  = 1  2    IA1  and IA0 = IB0 = IC0
 IC  1   2   I A2 

 IA = IA0 + IA1 + IA2 = 1.10


 IA1 + IA2 = 1.10 − IA0 = 1.0 …(i)

Also, IC = IA0 + IA1 + 2 IA2 = 1120 + 0.1

 IA1 + 2 + IA2 = 1120 …(ii)


Solving (i) and (ii),
IA1 = 1.0 pu and IA2 = 0

Then, IB = IA0 +  2 IA1 + IA2


= I B0 +  2 I A1 + I A2
= 0.1 + 1240 + 0
= 1 − 120 + 0.10

8. An alternator with internal voltage of I1 p.u and synchronous reactance of 0.4 p.u is connected by a
transmission line of reactance 0.1 p.u to a synchronous motor having synchronous reactance 0.35 p.u
and internal voltage of 0.852 p.u. If the real power supplied by the alternator is 0.866 p.u, then
( 1 − 2 ) is ___________ degrees. (Round off to 2 decimal places.)

(Machines are of non-salient type. Neglect resistances).


Key: (60)
Sol: E = 1 V
XS = j0.4 pu
R a = j0.1 pu
XS1 = j0.35 pu
P = 0.866 pu
Total power transferred,
Ef V 1 0.85
P= sin ( 1 − 2 )  0.866 = sin ( 1 − 2 )
Xeq 0.1 + 0.35 + 0.4

sin ( 1 − 2 ) = 0.866
1 − 2 = 60

© All rights reserved by Thinkcell Learning Solutions Pvt. Ltd. No part of this booklet may be reproduced or utilized in any form without the written permission.
|EE-2021|

9. Consider the boost converter shown. Switch Q is operating at 25 kHz with a duty cycle of 0.6. Assume
the diode and switch to be ideal. Under steady-state condition, the average resistance R in as seen by the
source is ___________  (Round off to 2 decimal places).
1mH

15V +

Q 100F 10

R in
Key: (1.6)
Sol: Given,
I s 1mH D

15V +

Q 100F 10 Vo

R in
V 
f operating = 25kHz and D = 0.6 We need to calculate average R in =  S 
 IS DC
VS 15 V 37.5
Vo = = = 37.5V , Io = o = = 8.75A
1 − D 1 − 0.6 Ro 10
Io 3.75
IS = = = 9.375A
1 − D 0.4
15
R in = = 1.6
9.375

Hence, the average resistance R in as seen by the source is 1.6.

10. If the input x(t) and output y(t) of a system are related as y ( t ) = max ( 0, x ( t ) ) , then the system is

(A) non-linear and time-invariant (B) linear and time-invariant


(C) non-linear and time-variant (D) linear and time-variant
Key: (A)
Sol: It is given that y ( t ) = Max ( 0,x ( t ))

y1 ( t ) = M ax 0, x1 ( t ) 
y2 ( t ) = M ax 0, x 2 ( t ) 

© All rights reserved by Thinkcell Learning Solutions Pvt. Ltd. No part of this booklet may be reproduced or utilized in any form without the written permission.
|EE-2021|

y3 ( t ) = Max 0, ( x1 ( t ) + x 2 ( t ) ) 
y 4 ( t ) = Max 0, x ( t ) 
y5 ( t ) = M ax 0, x ( t ) 

For additively we want y3 ( t ) = y1 ( t ) + y 2 ( t )

Homogeneity we want y 4 ( t ) = y5 ( t )

Let x (1) = 1 and x 2 ( t ) = −1 ( both D.C )

Then y1 ( t ) = Max ( 0,1) = 1


y2 ( t ) = Max ( 0, −1) = 0
y3 ( t ) = Max ( 0,0 ) = 0

y3 (t)  y1 ( t ) + y2 ( t ) So, system is nonlinear.

If input is x ( t − t o ) then y1 ( t ) = Max 0, x ( t − t o ) 


y ( t − t o ) = Max 0, x ( t − t o )

Since y1 ( t ) = y ( t1 − t o ) system is time invariant

So, system is nonlinear and time invariant.

11. Consider the buck-boost converter shown. Switch Q is operating at 25 kHz and 0.75 duty-cycle. Assume
diode and switch to be ideal. Under steady-state condition, the average current flowing through the
inductor is ____________A.

Iin Q
Io

20V +

1mH 100F 10

Key: (24)
Sol: Given, f operating = 25 kHz, D = 0.75 and converter is buck boost.

First of all we should check for continuity or discontinuity of converter

(1 − D ) (1 − 0.75) 10
2 2
R
LC = = = 1.25  10−5 or 12.5 H
2f 2  25  10 3

L ( given )  LC hence it is continuous mode of operation.

© All rights reserved by Thinkcell Learning Solutions Pvt. Ltd. No part of this booklet may be reproduced or utilized in any form without the written permission.
|EE-2021|

DVsc
Vo 1 − D
Io = = = 6A
R R
ID 6
 IL = = = 24A
1 − D 1 − 0.75
Hence, the average current flowing through the inductor is 24A.

12. Let A be a 10 × 10 matrix such that A 5 is a null matrix, and let I be the 10 × 10 identity matrix. The
determinant of A + I is ____________.
Key: (1)
Sol: Given A5 = 0
 A is Nilpotent matrix
 eigen values of A are all ‘0’
 eigen values of the matrix A + I are all equal to 0+1 = 1
 Determinant of (A+ I) is product of eigen values of (A+ I )
= 1 × 1× 1… ×1 (10 times) =1 (Using properties of eigen values)

13. In the given circuit, for voltage Vy to be zero, the value of  should be (Round off to 2 decimal places).

1 Vx 2 Vy 3

6V + 2A + Vx
− 4 −

Key: (–3.25)
Sol: In the following network we need to obtain value of  for which Vy is 0

1 Vx 2 Vy 3

6V + 2A + Vx
− 4 −

© All rights reserved by Thinkcell Learning Solutions Pvt. Ltd. No part of this booklet may be reproduced or utilized in any form without the written permission.
|EE-2021|

Writing Nodal equation at node X we have


Vx − 6 Vx − Vy Vx
+ + =0
1 2 4
 4Vx − 24 + 2Vx − 2Vy + Vx = 0
 7Vx − 2Vy = 24
24
If Vy = 0  Vx = V
7
Writing nodal equation at node y we have
Vy − Vx Vy −  Vx
+ −2=0
2 3
−Vx Vx
If Vy = 0 then − =2
2 3
 −3Vx − 2Vx = 12
 Vx ( 3 + 2 ) = −12
−12 −12
 3 + 2 =  2 = =3
Vx Vx

 7 
 2 =  −12   − 3
 24 
 2 = −3.5 − 3
−6.5
= = −3.25
2
  = −3.25

14. In the figure shown, self-impedance of the two transmission lines are 1.5j p.u each, and Zm = 0.5j p.u is
the mutual impedance. Bus voltage shown in the figure are in p.u. Given that   0, the maximum
steady-state real power that can be transferred in p.u from Bus-1 to Bus-2 is

Transmission
Line-1
Bus-1 Zm =0.5j Bus-2
E  Transmission E 0

Line-2

3E V E V
(A) 2 E V (B) (C) E V (D)
2 2

© All rights reserved by Thinkcell Learning Solutions Pvt. Ltd. No part of this booklet may be reproduced or utilized in any form without the written permission.
|EE-2021|

Key: (C)
Sol: We know,
I1 • L

I
M

L eq

I I2 L2

L1L2 − M 2
Where, Leq =
L1 + L2 − 2M

( jL1 )( jL2 ) − ( jM )


2

Or, jLeq =
( jL1 ) + ( jL2 ) − 2 ( jM )
X1s X 2s − X m2 1.5  1.5 − 0.52
Or, Xeq = = = 1 pu
X1s + X 2s − 2X m 1.5 + 1.5 − 2  0.5

E V EV
 Pmax = = =E V
X eq 1

15. In the circuit shown, the input Vi is a sinusoidal AC voltage having and RMS value of 230V  20%. The
worst-case peak-inverse voltage seen across any diode is __________V. (Round off to 2 decimal places).

R
Vi ~ C

Key: (390.32)
Sol: Given, Vi = 230  20%, and as given diagram
We have to calculated worst case peak inverse voltage seen across any diode.

© All rights reserved by Thinkcell Learning Solutions Pvt. Ltd. No part of this booklet may be reproduced or utilized in any form without the written permission.
|EE-2021|

2
 230  1.2  2 
( VD max )PIV = ( Vi )max + ( VC max )  + ( 230  1.2 ) = 2  ( 276 )
2
= 
2 2 2

 2 

(V )
D max PIV = 390.32V

The worst case peak-inverse voltage seen across any diode is 390.32V.

16. In the given circuit, for maximum power to be delivered to R L , its value should be _________ .
(Round off to 2 decimal places).

4mH

V ~  = 1k rad s 2 RL

0.5mF
Key: (1.414)
Sol: In the following network we need to obtain valved R L for maximum power transfer

4mH

 = 1k rad s ~ 2 RL

0.5mF

Since the load is purely resistance the condition for maximum power transfer is R L = Zth

Computation of Zth independent voltage source is short circuited.

j4
2

− j2 Zth

© All rights reserved by Thinkcell Learning Solutions Pvt. Ltd. No part of this booklet may be reproduced or utilized in any form without the written permission.
|EE-2021|

ZL = jL = j  1000  4  10−3 = j4


−1 −j
ZC = = = − j2
C 0.5  10−3  103
j4 j2 2 90
Zth = 2 || ( j4 − j2 ) = 2 || j2 = = = = 2 45
2 + j2 1 + j 2 45
R L = Zth = 2 45 = 2 = 1.414

17. Suppose the probability that a coin toss shown “head” is p, where 0 < p < 1. The coin is tossed
repeatedly until the first “head” appears. The expected number of tosses required is
1 1− p 1 p
(A) (B) (C) (D)
p2 p p (1 − p )
Key: (C)
Sol: Let p = Pr ( head )
q = Pr ( tail ) = 1 − p

Let X be a random variable denote number of tosses till to get one head then X = 1, 2, 3, … is a discrete
variable and
X = x P(x)
1 p
2 qp
3 qqp


 E ( x ) =  x.P ( x )
x =1

= 1 p + 2  qp + 3  q 2 p + ...
= p  1 + 2q + 3q 2 + ... = p  (1 − q )
−2

1
= p  p −2 =
p

© All rights reserved by Thinkcell Learning Solutions Pvt. Ltd. No part of this booklet may be reproduced or utilized in any form without the written permission.
|EE-2021|

18. The input impedance, Zin ( s ) , for the network shown is

1H
4

Zin ( s ) 6H 4H 5

(A) 6s + 4 (B) 7s + 4
25s 2 + 46s + 20 23s 2 + 46s + 20
(C) (D)
4s + 5 4s + 5

Key: (D)
Sol: In the following circuit we need to obtain input impedance.

4 1H

6H 4H 5

Zin

M L1 − M L2 − M

L2  M
L1

© All rights reserved by Thinkcell Learning Solutions Pvt. Ltd. No part of this booklet may be reproduced or utilized in any form without the written permission.
|EE-2021|

1H ( 6 − 1) H ( 4 − 1) H 5s 3s

6H 4H  1H  s

The original network in S domain becomes

4 3s
5s
• •

s 5

• •

Zin

3s 2 + 5s
Zin = ( 4 + 5s ) + s || ( 3s + 5 ) = 4 + 5s +
4s + 5
16s + 20 + 20s 2 + 25s + 3s 2 + 5s 23s 2 + 46s + 20
= =
4s + 5 4s + 5
23s 2 + 46s + 20
Zin =
4s + 5

E (s)
19. For the closed-loop system shown, the transfer function is
R (s)

R (s) + E (s) C (s)


G

GH G 1 1
(A) (B) (C) (D)
1 + GH 1 + GH 1+ G 1 + GH

© All rights reserved by Thinkcell Learning Solutions Pvt. Ltd. No part of this booklet may be reproduced or utilized in any form without the written permission.
|EE-2021|

Key: (D)
E (s)
Sol: We need to obtain for following block diagram
R (s)

R (s) + E (s)
G C (s)

C (s) G
= (standard negative feedback system)
R (s) 1 + GH

E (s ) = R (s ) − C (s ) H
GH
= R (s) − R (s )
1 + GH
 GH 
= R ( s ) 1 −
 1 + GH 
1 + GH − GH  1
= R (s)   = R (s )
 1 + GH  1 + GH

E (s) 1
 =
R (s) 1 + GH

20. A single-phase full-bridge inverter fed by a 325 V DC produces a symmetric quasi-square waveform
across ‘ab’ as shown.

Vab

325V
a
325V 0
   2 t
b
2
−325V

© All rights reserved by Thinkcell Learning Solutions Pvt. Ltd. No part of this booklet may be reproduced or utilized in any form without the written permission.
|EE-2021|

To achieve a modulation index of 0.8, the angle  expressed in degrees should be ________ (Round off
to 2 decimal places).
(Modulation index is defined as the ratio of the peak of the fundamental component of Vab to the applied
DC value).
Key: (51.07)
Sol: Given,
Vdc = 325V, m ( Need to achieve ) = 0.8

4Vdc
Vab =  cos n sin n ( t )
1,3,5 n

4  Vdc
( Vab )peak01 = cos 

4
( Vab01 )peak Vdc cos 
m= =  4
 0.8 = cos 
Vac Vdc 
 = 51.07
 To achieve a modulation index of 0.8, the angle  expressed in degrees should be 51.07 .

21. Let f ( x ) be a real-valued function such that f ' ( x 0 ) = 0 for some x 0  ( 0,1) , and f '' ( x )  0 for all
x  ( 0,1) . The f ( x ) has
(A) no local minimum in (0, 1) (B) one local maximum in (0, 1)
(C) exactly one local minimum in (0, 1) (D) two distinct local minima in (0, 1)
Key: (C)
Sol: Since f ' ( x 0 ) = 0  x 0 is called stationary point and x 0  ( 0,1)

f '' ( x )  0 for all x  ( 0,1)


 f '' ( x o )  0

 f(x) has local minimum at xo


i.e., exactly one local minimum in (0, 1)

22. Consider a continuous-time signal x ( t ) defined by x ( t ) = 0 for t  1, and x ( t ) = 1 − t for t  1. Let



the Fourier transform of x(t) defined as X ( ) =  x ( t )e
− jt
dt. . The maximum magnitude of X ( ) is
−

_________.

© All rights reserved by Thinkcell Learning Solutions Pvt. Ltd. No part of this booklet may be reproduced or utilized in any form without the written permission.
|EE-2021|

Key: (1)
Sol: Signal: CTFT
x(t) =1− t ; t 1
1
x(t):

−1 0 1 t

 width   1  2   2  
X ( ) = ( Area of triangle ) Sa 2    =   2 1 Sa   = Sa  
 4  2  2 2
 X ( 0 ) = Sa 2 ( 0 )
 X ( 0) = 1

23. ˆ m/s enters the region x  0 having a


One coulomb of point charge moving with a uniform velocity 10 X
magnetic flux density B = (10yxˆ + 10xyˆ + 10zˆ ) T. The magnitude for force on the charge at x = 0+ is
ˆ yˆ and zˆ are unit vectors along x-axis and z-axis, respectively).
_______ N. (x,
Key: (100)
Sol: The Lorentz magnetic force F due to change moving with velocity V in a magnetic field B is given by

(
F = q V*B )
Given, q = 1C, V = 10xˆ m sec, B = (10yxˆ + 10xyˆ + 10zˆ )

( ( ))
 F = 1 10xˆ  10yxˆ + 10xyˆ + 10z = 100x ( xˆ  yˆ ) + 100 ( xˆ  xˆ )

F = 100xzˆ + 100 ( yˆ )

At x = 0+
F = −100yˆ

 Magnitude of F = 100N

© All rights reserved by Thinkcell Learning Solutions Pvt. Ltd. No part of this booklet may be reproduced or utilized in any form without the written permission.
|EE-2021|

24. For the network shown, the equivalent Thevenin voltage and Thevenin impedance as seen terminals ‘ab’
is
3i1
2
• −+ a

5A 10 i1

• b

(A) 65 V in series with 15  (B) 35V in series with 2 


(C) 10 V in series with 12  (D) 50V in series with 2 
Key: (A)
Sol: In the following network we need to obtain Vth and R th
3i1
2
a
• −+

5A 10 i1

• b
Calculation of Vth

5A 0A 3i1
2
a
• −+
− + +

5A i1 = 5A Vth
10


• b

Vth − 0 − 3i1 − 10i1 = 0


Vth = 13i1 = 13  5 = 65V

Calculation of R th 3i1 2 IT
• −+

10 i1 + VT


© All rights reserved by Thinkcell Learning Solutions Pvt. Ltd. No part of this booklet may be reproduced or utilized in any form without the written permission.
|EE-2021|

(Current source is opened, dependent source is not disturbed)


VT − 2IT − 3i1 − 10i1 = 0 R th
VT = 2IT + 13i1 a
VT = 15IT ( i1 = IT ) 15
VT
IT
= 15 Vth ~ 65V

R th = 15
b
 Vth = 65V, R th = 15

25. Consider a power system consisting of N number of buses. Buses in this power system are categorized
into slack bus, PV buses and PQ buses for load flow study. The number of PQ buses is NL . The
balanced Newton-Raphson method is used to carry out load flow study in polar form. H, S, M and R are
sub-matrices of the Jacobian matrix J as shown below.
 P     H S
 Q  = J  V  where J =  M R 
     
The dimension of the sub-matrix M is
(A) N L  ( N − 1 + N L ) (B) ( N − 1)  ( N − 1 + NL )
(C) N L  ( N − 1) (D) ( N − 1)  ( N − 1 − NL )
Key: (C)
Sol: We know, Jacobian matrix is given by
 P P 
 H =  S=
V 
J= 
 Q Q 
M = R= 
  V

Where numerator part of each sub matrices talks about known quantities and denominator about
unknown.
Q
Here for M =

Q is known for PQ buses i.e., N L (given) and S is unknown for all buses except slack buses i.e., (N-1)

Hence, dimension of M should be N L  ( N − 1)

© All rights reserved by Thinkcell Learning Solutions Pvt. Ltd. No part of this booklet may be reproduced or utilized in any form without the written permission.
|EE-2021|

26. The power input to a 500V, 50 Hz, 6-pole, 3-phae induction motor running at 975 RPM is 40 kW. The
total stator losses are 1 kW. If the total friction and windage losses are 2.025 kW, then the efficiency is
__________%.
Key: (90)
Sol: Given,
P=6 
V = 500V, 

f = 50Hz 
 m = ?
Pi = 40kW 
N r = 975 rpm 

f and W loss = 2.025 kW 

Synchronous speed,
120f 120  50
NS = =
P 60
NS = 1000 rpm

NS − Nr 1000 − 975
S= = = 0.025
NS 1000
Pgm = (1 − S) Pi = (1 − 0.025 )  ( Pin − stator loss )
= 0.975  (40 − 1)

Pgm = 38.025 kW

Motor efficiency,
Pout Pgm − Pmech 38.025 − 2.025
m = = =
Pin Pin 40
m = 0.9 = 90%

27. A 1 C point charge is held at the origin of a Cartesian coordinate system. If a second point charge of
10 C is moved from (0, 10, 0) to (5, 5, 5) and subsequently to (5, 0, 0), then the total work done is
________ mJ. (Round off to 2 decimal places).
1
Take = 9 109 in SI units. All coordinates are in meters.
4o
Key: (9)
Sol: Work done depends on initial and final position
Initial point → ( 0,10,0 ) distance from origin = 10

© All rights reserved by Thinkcell Learning Solutions Pvt. Ltd. No part of this booklet may be reproduced or utilized in any form without the written permission.
|EE-2021|

Final point → ( 5,0,0 ) distance from origin = 5


A
WAB = −q 2 ( VA − VB ) 10
q1 1  10−6  9  109 origin
VA = = = 9  102
4o rA 10
q1 1  10−6  9  109
VB = = = 18  102 5
4o rB 5
B
Now = 10 10−6 9 102  = 90 10−4 = 9 10−3 J or 9 mJ

28. A counter is constructed with three D flip-flops. The input-output pairs are named ( D0 ,Q0 ) , ( D1 ,Q1 )
and ( D2 ,Q2 ) where the subscript 0 denotes the least significant bit. The output sequence is desired to be the
Gray-code sequence 000, 001, 011, 010, 110, 111, 101 and 100 repeating periodically. Note that the bits are
listed in the Q2 Q1Q0 format. The combinational logic expression for D1 is

(A) Q2 Q0 + Q1Q0 (B) Q2 Q1Q0

(C) Q2Q1 + Q2Q1 (D) Q2 Q0 + Q1Q0


Key: (D)
Sol: The question is about counter design based on given information we can have the following table.
Present state Next state Flip flop inputs
Q2 Q1 Q0 Q +2 Q1+ Q 0+ D2 D1 D0

0 0 0 0 0 1 0 0 1
0 0 1 0 1 1 0 1 1
0 1 0 1 1 0 1 1 0
0 1 1 0 1 0 0 1 0
1 0 0 0 0 0 0 0 0
1 0 1 1 0 0 1 0 0
1 1 0 1 1 1 1 1 1
1 1 1 1 0 1 1 0 1
D1 ( Q2 ,Q1 ,Q0 ) = m (1, 2,3,6 )
Q1Q0
D1 = Q2Q0 + Q1Q0 Q2 Q1Q 0 Q1Q 0 Q1Q0 Q1Q 0
Q 2 Q0
Q2 1 1 1 Q1Q 0

Q2 1

© All rights reserved by Thinkcell Learning Solutions Pvt. Ltd. No part of this booklet may be reproduced or utilized in any form without the written permission.
|EE-2021|

The causal signal with z-transform z2 ( z − a )


−2
29. is (u[n] is the unit step signal)

(A) a 2n u  n  (B) n −1a n u  n  (C) n 2 a n u  n  (D) ( n + 1) a n u  n 


Key: (D)
z2 1
Sol: Given X ( z ) = =
(z − a) (1 − az )
2 −1 2

1
We know,  ( n + 1) a n u ( n )( standard z-transform )
(1 − az ) −1 2

30. A belt-driven DC shunt generator running at 300 RPM delivers 100 kW to 200V DC grid. It continues to
run as a motor when the belt breaks, taking 10 kW from the DC grid. The armature resistance is
0.025 , field resistance is 50 , and brush drop is 2V. Ignoring armature reaction, the speed of the
motor is ____________RPM. (Round off to 2 decimal places).
Key: (275.18)
Sol: Given,
N = 300 rpm 
P0 = 100 kW 

V = 200V 
 N=?
R a = 0.025 
R sh = 50 

Brush drop = 2V 

P0 100  103
Line current, IL = = = 500A
V 200
Vbus = 200V
V 200
Ish = = = 4A
R sh 50 IL

Ia = I L + Ish Ish
Ia = 500 + 4
Ra 0.025
Ia = 504A R sh = 50
Ia
Apply KVL
Eg
E g − Ia R a − B.D = VBus
E g = Ia R a + B.D + VBus
= 504  0.025 + 2 + 200
Eg = 214.6V

If belt brokes,

© All rights reserved by Thinkcell Learning Solutions Pvt. Ltd. No part of this booklet may be reproduced or utilized in any form without the written permission.
|EE-2021|

P = 10  103
P
I L = = 50A
V
V 200
Ish = = = 4A
R sh 50

Ia = IL − Ish = 50 − 4
Ia = 46A
By KVL,
V − Ia R a − BD = E b
200 − 46  0.025 = E b
E b = 196.85V
Eb
N

‘  ‘ is constant since current through shunt field winding is constant
Nm Em
 =
Ng Eg

Em 196.85
Nm =  Ng =  300
Eg 214.6
N m = 275.18 rpm

IB
31. A three-phase balanced voltage is applied to the load shown. The phase sequence is RYB. The ratio
IR
is ____________.

IR

− j10

IB j10

IY

© All rights reserved by Thinkcell Learning Solutions Pvt. Ltd. No part of this booklet may be reproduced or utilized in any form without the written permission.
|EE-2021|

Key: (1)
IB IR
Sol: In the following network we need to obtain R
IR
− j10
Since the source is balanced
VRY = V 0 B
VYB = V −120 j10
VBR = V 120 IB
B
VRB −VBR −V 120 V
IR = = = = 30
− j10 − j10 − j10 10
Y
VYB V −120 V
IY = = = −210 IY
j10 j10 10
IR + IY + IB = 0

V V  −V −V
 IB = − IR + I y  =  30 + −210 = 1 30 + 1 −210 = 90
10 10  10 10
V V
IR = 30 =
10 10
−V V
IB = 90 =
10 10
IB
=1
IR

32. In the circuit, switch ‘S’ is in the closed position for a very long time. If the switch is opened at time
t = 0, then i L (t) in amperes, for t  0 is

t=0
S
1

10V 4 30V iL 0.5H

(A) 8e−10 (B) 10 (


(C) 10 1 − e−2t ) (D) 8 + 2e−10t

© All rights reserved by Thinkcell Learning Solutions Pvt. Ltd. No part of this booklet may be reproduced or utilized in any form without the written permission.
|EE-2021|

Key: (D)
Sol: In the following network we need to obtain i L ( t ) for t  0

t=0

1
Network in steady state
Switch is closed
10V 4
30V 0.5H iL ( t )
at t = 0−
Inductor is short circuited

1 Network is in steady state


+ − at t =  Switch is open
10V
i L ( 0− ) = 10A
10V 4
30V Inductor is short circuited

Network for t  0
1 is to be referred
R th
Independent voltage
10 + 30 source to be shorted
10V 4
30V iL (  ) = = 8A
4 +1

1

4

R th = 1 + 4 = 5

L 0.5 1
Time constant = = =
R th 5 10

i L ( t ) = i L (  ) + i L ( 0− ) − i L (  ) e− t  ; t  0 = 8 + 10 − 8 e −10t = 8 + 2e −10t ; t  0

© All rights reserved by Thinkcell Learning Solutions Pvt. Ltd. No part of this booklet may be reproduced or utilized in any form without the written permission.
|EE-2021|

33. An air core radio-frequency transformer as shown has a primary winding and a secondary winding. The
mutual inductance M between the windings of the transformer is ___________ H. (Round off to 2
decimal places). M

100 kHz ~ 7.3 Vp − p

22 5.0 Vp − p

Key: (51.1) M
Sol: Given,

7.3 Vp − p
100 kHz ~

22 5.0Vp − p

I Primary

We need to calculate mutual inductance M between the windings of the transformer.


Secondary is open circuited,
Isec = 0A

 ( 7.3)p − p = MIprimary
5 22 5
 7.3 = 2    100  103  M  = 2   100  103  M   M = 51.1 H
22 7 22
Hence, the mutual inductance M between the windings of the transformer is 51.1 H.

© All rights reserved by Thinkcell Learning Solutions Pvt. Ltd. No part of this booklet may be reproduced or utilized in any form without the written permission.
|EE-2021|

p q 
34. Let p and q be real numbers such that p2 + q 2 = 1. The eigen values of the matrix   are
q −p 
(A) j and –j (B) 1 and 1 (C) 1 and –1 (D) pq and –pq
Key: (C)
Sol: Characteristic equation of the matrix is
 2 −   ( p − p ) + ( −p2 − q 2 ) = 0
  2 − 1 = 0   = −1,1 ( since p 2 + q 2 = 1)

( H int :  2
−   ( trace ) + ( determinant of matrix ) )

35. Let p ( z ) = z3 + (1 + j) z 2 + ( 2 + j) z + 3, where Z is a complex number. Which one of the following is


true?
(A) All the roots cannot be real
(B) conjugate {p(z)} = p(conjugate {z}) for all z
(C) The sum of the roots of p(z) = 0 is a real number
(D) The complex roots of the equation p(z) = 0 come in conjugate pairs
Key: (A)
Sol: Let z = j be a complex number
 z = −j

p ( z ) = j3 + (1 + j) j2 + ( 2 + j) j + 3 = − j − 1 − j + 2j − 1 + 3 = 1
 p(z) = 1

And p ( z ) = p ( − j) = ( − j) + (1 + j)( − j) + ( 2 + j)( − j) + 3


3 2

= j − 1 − j − 2j + 1 + 3 = 3 − 2j

 p( z)  p ( z )
 Option ‘B’ is not true.
Eg: 1) Let p ( z ) = z3 − jz 2 + z − j has a 3 complex roots z = − j, j, j
2) Let P ( z ) = z3 − z 2 + z − 1 has 2 complex roots − j, j and one real root

∴ From the above two examples, sum of the roots need not be a real number and all the roots cannot
be real.
∴ Option 'A' is the correct answer.

© All rights reserved by Thinkcell Learning Solutions Pvt. Ltd. No part of this booklet may be reproduced or utilized in any form without the written permission.
|EE-2021|

14.4
36. Consider a closed-loop system as shown. G p ( s ) = is the plant transfer function and
s (1 + 0.1s )
G c ( s ) = 1 is the compensator. For a unit-step input, the output response has damped oscillations. The
damped natural frequency is ____________rad/s. (Round off to 2 decimal places).

R (s) + E (s) C (s)


Gc (s ) Gp (s)

Key: (10.90)
Sol: The given block diagram is as follows

R (s) + E (s)
Gc (s) = 1 Gp (s) =
14.4 C (s)
− s (1 + 0.1s )

It is given that for unit step input response has damped oscillation we need to obtain damped natural
frequency.
The closed loop transfer function of above block diagram is
14.4
C (s) G (s ) G (s ) s (1 + 0.1s ) 14.4
= = =
R ( s ) 1 + G c ( s ) G p ( s ) 1 + 14.4 s (1 + 0.1s ) + 14.4
s (1 + 0.1s )
14.4 144 2n
= = =
s + 0.1s 2 + 14.4 s 2 + 10s + 144 s 2 + 2n s + 02

By comparison 2n = 144  n = 12 rad sec

10 10
2n = 10   = = = 0.416
2n 24

Damped natural frequency: d = n 1 −  = 12 1 − ( 0.416 ) = 10.90 rad sec


2

© All rights reserved by Thinkcell Learning Solutions Pvt. Ltd. No part of this booklet may be reproduced or utilized in any form without the written permission.
|EE-2021|

37. Two single-cover power cables have total conductor resistances of 0.7  and 0.5 , respectively, and
their insulation resistance (between core and sheath) are 600 M and 900 M respectively. When the
two cables are joined in series, the ratio of insulation resistance to conductor resistance is _______ 106.
Key: (300)
Sol: RC( total) = RC1 + RC2 (series ) = 0.7 + 0.5 = 1.2

600  900
R ins total = = 360  106 
600 + 900
R ins( total) 360 106
= = 300  106
R C( total) 1.2

38. An 8-pole, 50 Hz, three-phase, slip-ring induction motor has an effective rotor resistance of 0.08  per
phase. Its speed at maximum torque is 650 RPM. The additional resistance per phase that must be
inserted in the rotor to achieve maximum torque at start is ___________ . (Round off to 2 decimal
places). Neglect magnetizing current and stator leakage impedance. Consider equivalent circuit
parameters referred to stator.
Key: (0.52)
Sol: Given,
P = 8, 
f = 50Hz, 

R = ?
R 2 = 0.08  ph  add
N Tm = 650 rpm 
120f 120  50
Synchronous speed, NS = = = 750 rpm
P 8
Slip at maximum torque,
NS − NTm 750 − 650
STm = = = 0.133
NS 750
We also have,
R '2 R' 0.08
STm =  X'2 = 2 = = 0.6
X '2 STm 0.133
For producing maximum torque at starting
STm = 1

R '2( new )
 = 1  R '2( new ) = X '2 = 0.6
X '2
R ext = 0.6 − R '2 = 0.6 − 0.08
R ext = 0.52

© All rights reserved by Thinkcell Learning Solutions Pvt. Ltd. No part of this booklet may be reproduced or utilized in any form without the written permission.
|EE-2021|

39. Consider a large parallel plate capacitor., The gap d between the two plates is filled entirely with
dielectric slab of relative permittivity 5. The plates are initially charged to a potential different of V volts
and then disconnected from the source. If the dielectric slab is pulled out completely, then the ratio of
the new electric field E 2 in the gap to the original electric field E1 is _________.
Key: (5)
Sol: On removal of voltage source, charge on the capacitor will remain constant.
o 5A
Initially, Q1 = .v1 ( Q = CV ) …(1)
d
After removing dielectric slab and voltage source
o .A
Q1 = .V2 …(2)
d
V1 and V2 are potential between plates

V E V E1 → original electric field


Electric field E =  1 = 1
d E 2 V2 E 2 → new electric field

o 5.A  .A
Since (1) = (2), V1 = o V2
d d
V1 1
=
V2 5

E1 1
 = or ratio of new electric field strength to original electric field strength = 5
E2 5

40. Two generators have cost functions F1 and F2 . Their incremental-cost characteristics are
dF1
= 40 + 0.2P1
dP1
dF2
= 32 + 0.4P2
dP2
They need to deliver a combined load of 260 MW. Ignoring the network losses, for economic operation,
the generations P1 and P2 (in MW) are
(A) P1 = P2 = 130 (B) P1 = 160, P2 = 100
(C) P1 = 140, P2 = 120 (D) P1 = 120, P2 = 140
Key: (B)
dF1 dF2
Sol: =
dP1 dP2
40 + 0.2P1 = 32 + 0.4P2
40 − 32 = 0.4P2 − 0.2P1

© All rights reserved by Thinkcell Learning Solutions Pvt. Ltd. No part of this booklet may be reproduced or utilized in any form without the written permission.
|EE-2021|

8 = 0.4P2 − 0.2P1 …(1)


P1 + P2 = 260 MW …(2)
 P1 − 2P2 = −40 …(1)
 P1 + P2 = 260 …(2)
Solve (1) and (2) for finding the values of P1 and P2
−3P2 = −300
P2 = 100MW
P1 = 160MW

41. A 3-Bus network is shown. Consider generators as ideal voltage sources. If rows 1, 2 and 3 of the YBus
matrix correspond to Bus 1, 2 and 3, respectively then YBus of the network is

~ Bus-2

Z2 = j
Z1 = j
Bus-1 Bus-3
Z3 = j

~ Z4 = j ~
 1 1 1 
− 2 j 4 j j
4 
 −4 j 2 j 2 j   
(A)  2 j −4 j 2 j   1 1 1 
(B) j − j j
 4 2 4 
 2 j 2 j −4 j  
 1j 1 1
j − j
 4 4 2 

 3 1 1 
− 4 j 4 j j
4 
 −4 j j j   

(C)  j −4 j j  (D)  1 3
j − j
1 
j
 4 4 4 
 j j −4 j  
 1j 1 3
j − j
 4 4 4 
Key: (D)

© All rights reserved by Thinkcell Learning Solutions Pvt. Ltd. No part of this booklet may be reproduced or utilized in any form without the written permission.
|EE-2021|

Sol:
4

I4
~ V4
j1

3
1 I3

I1 V2
V1 V3
~ j1 ~
I2

All voltage sources are ideal


I1 + I3 + I4 = I2 …(1)

I1 = ( V1 − V2 )( − j1)

I1 = ( − j1) V1 + ( j1) V2 …(2)

I2 = V2 ( − j1) …(3)

I3 = ( − j1) V2 + ( j1) V3 …(4)

I 4 = ( j1) V2 − ( j1) V4 …(5)

I 4 = ( V4 − V2 )( − j1)
I 4 = ( j1) V2 − ( j1) V4 − ( 5)

From equation (1) and (3)


I1 + I3 + I4 = V2 ( − j1)

V2 = ( j1) ( − j1) V1 + ( j1) V2 + ( j1) V2 + ( − j1) V3 + ( j1) V2 − ( j1) V4 

V2 = V1 − 3V2 + V3 + V4
4V2 = V1 + V3 + V4

1 1 1
V2 = V1 + V3 + V4 …(6)
4 4 4

© All rights reserved by Thinkcell Learning Solutions Pvt. Ltd. No part of this booklet may be reproduced or utilized in any form without the written permission.
|EE-2021|

1 1 1 
I1 = ( j1) V1 + ( j1)  V1 + V3 + V4 
4 4 4 
3 1 1
I1 = − j V1 + j V3 + j V4 − ( 7 )
4 3 4
 1 1 1 
I3 = ( − j1) V3 − V1 − V3 − V4 
 4 4 4 
1 3 1
I3 = j V1 − j V3 + j V4 …(8)
4 4 4
1 1 1 
I4 = ( j1)  V1 + V3 + V4  − ( j1) V4
4 4 4 
1 1 3
I4 = j V1 + j V3 − j V4 …(9)
4 4 4
 3 1 1 
− 4 j 4 j 4
j
 1 
I   V1 
I  =  1 j − 3 j 1 j  V 
 2  4 4 4  
3

 I3    V

 1j 1 3  4
j − j
 4 4 4 
YBus

42. Which one of the following vector functions represents a magnetic field B?
ˆ yˆ and zˆ are unit vectors along x-axis, y-axis and z-axis respectively)
(x,
(A) 10xxˆ + 20yyˆ − 30zzˆ (B) 10yxˆ + 20xyˆ − 10zzˆ
(C) 10xxˆ − 30zyˆ + 20yzˆ (D) 10zxˆ + 20yyˆ − 30xzˆ
Key: (A)
Sol: By Maxwell’s equation we know that divergence of magnetic field in zero
.B = 0
Option 1: .B = 10 + 20 − 30 = 0
Option 2: .B = 0 + 0 − 10 = −10
Option 3: .B = 10 + 0 + 0 = 10
Option 4: .B = 0 + 20 − 0 = 20
  
.B = Bx + By + Bz
x y y
Thus option (A) is correct answer.

© All rights reserved by Thinkcell Learning Solutions Pvt. Ltd. No part of this booklet may be reproduced or utilized in any form without the written permission.
|EE-2021|

43. The state space representation of a first-order system is given as


x = −x + u
y=x
Where, x is the state variable, u is the control input and y is the controlled output. Let u = –Kx be the
control law, where K is the controller gain. To place closed-loop pole at –2, the value of K is _______.
Key: (1)
Sol: It is given that
x = −x + u
y=x
u = − kx
We need to obtain value of such that closed loop pole will be at s = –2.
Closed loop pole is roots of characteristic equation,
Characteristic equation in terms of state space is sI − A = 0

x = − x + u = − x − kx ( u = −kx )

 sX ( s ) = −X ( s ) − kX ( s )  x = x ( −k − 1)
 X ( s ) s + 1 − k  = 1  x = Ax + BU

By comparison A = −k − 1
Characteristic equation: sI − A = 0

s − ( −k − 1) = 0
s + k +1 = 0
s + k +1 = 0 ( Determinant of a constant is constant )
k +1 = 2 ( closed loop pole is k + 1 and if should be at 2 )
k =1

44. The waveform shown in solid line is obtained by clipping a full-wave rectified sinusoid (shown dashed).
The ratio of the RMS value of the full-wave rectified waveform to the RMS value of the clipped
waveform is _______(Round off to 2 decimal places).

Vm

0.707Vm

0  2 3 t
 3
4 4
© All rights reserved by Thinkcell Learning Solutions Pvt. Ltd. No part of this booklet may be reproduced or utilized in any form without the written permission.
|EE-2021|

Key: (1.21)
Sol: In this case we need to obtain the ratio of r.m.s values of 2 wave forms.

Vu ( unclipped waveform )
VC ( clipped waveform )

Vm Vm
.... 2

0  2 3
0 
 3
T
1 o 4 4
 Vu rms  =  Vu 2 dt
2 To

(V ) 1
2
=  V dt
2
T0 0 Crms C

To 0
1
=  ( Vm sin t ) dt
2
1 
4 3 4 2 
0  Vm 
=  m +  +  Vm sin 2 tdt 
2 2

  0
V sin tdt   dt
 4  2 3 4 
Vm2
(1 − cos 2t ) dt
2 0
=
Vm2   4 1 − cos 2t 1
3 4 / 4
 1 − cos 2t  
=  dt +  dt +   dt  
  0 3 4   
Vm2    sin 2t  
 2 2 4 2
= ( t ) −   
2  0  2 0  Vm2  4 1
3 4

0 =  (1 − cos 2t ) dt +  dt 
Vm2    0 2  4 
(  − 0 ) − ( sin 2 − sin 0 )
1
= 
2  2  Vm2   4  sin 2t   4 1 3 4 
2 = ( t )0 −   + ( t ) 4 
= m
V    2 0 2 
2
V  1  
Vm2  1  3   
Vu rms = m = 0.707Vm =  4 − 2  sin 2 − sin 0  + 2  4 − 4  
2     
V2   1  
= m − + 
 4 2 4
Vm2   1  Vm2   − 1 
= − =  
  2 2    2 
  −1
= Vm2  
 2 
Vu rms 0.707Vm
= = 1.21   −1 
VCrms 0.583 Vm VCrms = Vm2   = 0.5838 Vm
 2 

© All rights reserved by Thinkcell Learning Solutions Pvt. Ltd. No part of this booklet may be reproduced or utilized in any form without the written permission.
|EE-2021|

45. Two discrete-time linear time-invariant systems with impulse responses


h1  n  =   n − 1 +   n + 1 and h 2  n  =   n  +   n − 1 are connected in cascade, where   n  is the
Kronecker delta. The impulse response of the cascaded system is
(A)   n    n − 1 +   n − 2   n + 1 (B)   n − 2 +   n + 1

(C)   n − 2 +   n − 1 +   n  +   n + 1 (D)   n − 1   n  +   n + 1   n − 1

Key: (C)
Sol: It is given that
h1 ( n ) =  ( n − 1) +  ( n + 1)
h 2 ( n ) =  ( n ) +  ( n − 1)

Then h ( n ) = h1 ( n ) * h 2 ( n )


=  ( n − 1) *   
 ( n ) +  ( n − 1) +  ( n + 1) * 
 ( n ) +  ( n − 1) 
 ( n − 1) *  ( n ) + 
=   ( n − 1) *  ( n − 1) + 
 ( n + 1) *  ( n ) + 
 ( n + 1) *  ( n − 1)
=  ( n − 1) +  ( n − 2 ) +  ( n + 1) +  ( n )   ( n − n o ) *  ( n − n1 ) =   n − ( n 0 + n1 ) 
 
=  ( n − 2 ) +  ( n − 1) +  ( n ) +  ( n + 1)

2.2  1 + T1s 
46. In the given figure, plant G p ( s ) = and compensator G c ( s ) = K  .
(1 + 0.1s )(1 + 0.4s )(1 + 1.2s )  1 + T2s 
The external disturbance input is D(s). It is desired that when the disturbance is a unit step, the steady-
error should not exceed 0.1 unit. The minimum value of K is ___________. (Round off to 2 decimal
places).
D (s)
+ +
R (s) E (s) C (s)
Gc (s ) Gp (s)
− +

Key: (-10.45)

© All rights reserved by Thinkcell Learning Solutions Pvt. Ltd. No part of this booklet may be reproduced or utilized in any form without the written permission.
|EE-2021|

Sol: The given block diagram is


D (s)
+ +
R (s) E (s) C (s)
Gc (s ) Gp (s)
− +

2.2  1 + T1s 
Gp (s) = , Gc (s ) = k  
(1 + 0.1s )(1 + 0.4s )(1 + 1.2s )  1 + T2s 
When D(s) is unit step the steady state error should be at maximum 0.1, we need to obtain minimum
value of k.
Since there are 2 inputs (R(s), D(s)), we want E(s) due to D(s), so R(s) should be made 0
E (s) Gp (s)
=
D (s) 1 + G pGc (s )
−G p ( s )  1
E (s) =  D (s ) = 
s (1 + G p ( s ) G 0 ( s ) )  s

−G p ( s )
Steady sate error e (  ) = limsE ( s ) = lim
s →0 s →0 1 + G p (s ) Gc (s )

−2.2

 e (  ) = lim
(1 + 0.1s )(1 + 0.4s )(1 + 1.2s ) =
−2.2
s →0  1 + T1s  2.2 1 + 2.2k
1+ k 
 1 + T2s  (1 + 0.1s )(1 + 0.4s1 + 1.2s )
We need e (  )  0.1

−2.2
 0.1 0.22k  −2.3
1 + 2.2k
−2.3
 −2.2  0.1 + 0.22k k
0.22
 −2.3  0.22k k  −10.45
Minimum value of k is –10.45.

© All rights reserved by Thinkcell Learning Solutions Pvt. Ltd. No part of this booklet may be reproduced or utilized in any form without the written permission.
|EE-2021|

47. In the open interval (0, 1), the polynomial p ( x ) = x 4 − 4x 3 + 2 has

(A) two real roots (B) three real roots


(C) one real root (D) no real roots
Key: (C)
Sol: p ( 0 ) = z  0 and p (1) = 1 − 4 + 2 = −1  0

 One of the roots lies between 0 and 1

Signs of p ( x ) : + − +  two sign changes in p ( x )

Signs of p ( −x ) : + + +  no sign changes in p ( −x )

∴ p(x) has at most two positive roots and no negative root. We know that complex roots occur in pairs
and p(x) is a 4th degree polynomial also,
p ( 3) = 81 − 108 + 2  0 and
p ( 4 ) = 256 − 256 + 2  0  root lies between 3 and 4

Hence p(x) has one real root in (0, 1)


∴ Option (C) is the correct answer.

48. In the given circuit, the value of capacitor C that makes current I = 0 is ______ F.

I 10 j5 j5

10V ~  = 5k rad s j5 C

© All rights reserved by Thinkcell Learning Solutions Pvt. Ltd. No part of this booklet may be reproduced or utilized in any form without the written permission.
|EE-2021|

Key: (20)
Sol: In the following network we need to obtain value of C for which I = 0
10 j5 j5
I

10V ~  = 5k rad s j5 C

Zx Z

I 10 j5 j5

10V ~  = 5k rad s j5 C

V
If Z' =  then Zx =  and hence I = = 0A
Zx

 1 
j5  j5 + 
 1   jC 
Z' = j5 ||  j5 + =
 jC  j5 + j5 + 1
jC
1
For Z' =  we need j5 + j5 + =0
jC
+j
 j10 =
C
1 1
C= = = 20 F
10 10  5  103
For current I = 0 we need C = 20 F.

© All rights reserved by Thinkcell Learning Solutions Pvt. Ltd. No part of this booklet may be reproduced or utilized in any form without the written permission.
|EE-2021|

49. In the BJT circuit shown, beta of the PNP transistor is 100. Assume VBE = −0.7V. The voltage across
R C will be 5V when R 2 is __________ k. (Round of to 2 decimal places).

1.2 k
R1 RE
4.7 k

12V

R2 RC
3.3 k

Key: (17.53)
Sol:

4.7 k R1 RE 1.2 k

12V

R2 RC 3.3 k

VC 5
Given VC = 5V, So, IC = = mA
R C 3.3

IC   + 1  101 5
IE = =  IC =  mA = 1.53 mA
    100 3.3
VE = 12V − 1.53  1.2
VE = 10.164V
Given, VBE = −0.7V
VB − VE = −0.7V

© All rights reserved by Thinkcell Learning Solutions Pvt. Ltd. No part of this booklet may be reproduced or utilized in any form without the written permission.
|EE-2021|

 VB = VE − 0.7V = 10.164V − 0.7V = 9.464V


By applying Thevenin’s equivalent at the input bias network.
R2
VB = VTh =  VCC
R1 + R 2
R2
9.464V =  12V
4.7k + R 2
4.7k + R 2 12
=
R2 9.464
 4.7k  12
1 + =
 R 2  9.464
4.7k 12
= − 1 = 0.267
R2 9.464
 R 2 = 17.53k

Suppose the circle x3 + y2 = 1and ( x − 1) + ( y − 1) = r 2 intersect each other orthogonally at the point
2 2
50.
(u, v ). The u + v = _________,
Key: (1)

Let f ( x, y ) = x 2 + y2 = 1......(1) and g ( x, y ) = ( x −1) + ( y −1) = r 2 ……(2)


2 2
Sol:

f

Then
dy
= x = −2x = − x is the slope of curve (1)
dx f 2y y
y

g

And
dy
= x = −2 ( x − 1) = − ( x − 1) is the slope of curve (2)
dx g 2 ( y − 1) y −1
y
For orthogonal curves, product of slopes is –1
x (x − 1)
  = −1
y (y − 1)
 x 2 − x = − y2 + y
 x 2 + y2 = x + y
 x + y =1 ( x 2 + y 2 = 1)

Since (1) and (2) intersect at (u, v)


u+v=1

© All rights reserved by Thinkcell Learning Solutions Pvt. Ltd. No part of this booklet may be reproduced or utilized in any form without the written permission.
|EE-2021|

51. A 16-bit synchronous binary up-counter is clocked with a frequency fCLK . The two most significant bits
are OR-ed together to form an output Y. Measurement show that Y is periodic, and the duration for
which Y remains high in each period is 24ms. The clock frequency f CLK is ______ MHz.
(Round off to 2 decimal places).
Key: (2.048)
Sol: From the given information we can draw the following block diagram

16 bit binary up counter


( synchronous type )
Clock

Q15 Q14 Q13 Q Q 2 Q1 Q 0


12

It is given that Y = 1 for 24 m.sec in every period of Y. We need to obtain the clock frequency.
Since it’s a 16-bit synchronous binary up counter, when it will receive clocks it will change its states
from [00…000 to 11…111] i.e., total 216 number of distinct states in one counting cycle.
Y will be 0 only when Q15 and Q14 both are zero simultaneously else Y will be 1.

So out of 216 number of states, the number of states when Q15 = Q14 = 0 is 214 (i.e., Q15 = Q14 = 0, but
rest of the 14 bits Q13 to Q0 can vary minimum to maximum).

Total 216 number of states


Out of 216 states in 214 states Y= 0

(
Out of 216 states 216 − 214 states Y = 1 )
(2 16
− 214 ) Tclk = 24 m.sec

24  10−3
Tclk = = 488  10−9 sec = 488 n.sec
216 − 214
1 1
f clk = = = 2.048 MHz
Tclk 488  10−9

© All rights reserved by Thinkcell Learning Solutions Pvt. Ltd. No part of this booklet may be reproduced or utilized in any form without the written permission.
|EE-2021|

Vo ( s )
52. The bode magnitude plot for the transfer function of the circuit is as shown.
Vi ( s )

The value of R is ____________ . (Round off to 2 decimal places).


Bode diagram
R 1mH

Magnitude [dB]
26dB
Vi 250F Vo

102 103 104 1


 = 2000rad/s
Key: (0.1)
Sol: For the following RLC circuit, the gain at frequency 2000rad/sec is given 26dB. We need to obtain value
of R
R 1mH

Vi Vo
250F

When the operating frequency is 2000 rad/sec


ZL = jL = j  2000  1  10−3 = j2
−j −1
Zc = = = − j2
C 200  250  10−6
In general
ZC
Vo ( j) = Vi ( j)
R + ZL + ZC

Vo ( j) − j2
 = ( at  = 2000 r sec )
Vi ( j) R + j2 − j2
Vo ( j) 2
 = ...(1)
Vi ( j) R

Vo ( j)
It is given that 20log = 26
Vi ( j)

© All rights reserved by Thinkcell Learning Solutions Pvt. Ltd. No part of this booklet may be reproduced or utilized in any form without the written permission.
|EE-2021|

Vo ( j) 26
 = 10 20 ( at  = 2000 rad sec )
Vi ( j)
Vo ( j)
 = 19.95 ...(2)
Vi ( j)

2 2
Equating equation (1) and (2) = 19.95  R =  R = 0.1
R 19.95

53. In the circuit shown, a 5V Zener diode is used to regulate the voltage across load R 0 . The input is an
unregulated DC voltage with a minimum value of 6V and a maximum value of 8V. The value of R S is
6 . . The zener diode has a maximum rated power dissipation of 2.5 W. Assuming the zener diode to be
ideal, the minimum value of R 0 is _______ .
Rs

Vi Vz R0

Key: (10)
Sol:
RS Ideal zener diode characteristic
IL
IZ −VZ
IS
Vi VZ R0 0

−I Z
For ideal Zener diode Iz min = Iknee = 0mA
Given, PZmax = IZmax VZ
PZ max 2.5
I z max = = = 0.5A
VZ 5
VZ
R 0( min ) = =?
I L( max )
IS = I Z + I L

© All rights reserved by Thinkcell Learning Solutions Pvt. Ltd. No part of this booklet may be reproduced or utilized in any form without the written permission.
|EE-2021|

IS( max ) = I Zmin + I L( max )


Vi( max ) − VZ
= 0 + IL( max )
RS
8−5 3
I L( max ) = = = 0.5A
6 6
5
R 0( min ) = = 10
0.5
 R 0( min ) = 10

54. Let ( −1 − j) , ( 3 − j) , ( 3 + j) and ( −1 + j) be the vertices of a rectangle C in the complex plane. Assuming
dz
that C is traversed in counter-clockwise direction, the value of the contour integral  z ( z − 4)
C
2
is

− j j j
(A) 0 (B) (C) (D)
8 16 2
Key: (B)
Sol: Let A ( −1, −1) = −1 − j; B ( 3, −1) ; D ( 3,1) and E ( −1,1) be the vertices of rectangle

z 2 ( z − 4 ) = 0  z = 0, 4 Y

 1  E 1 D
 
dz  z − 4 dz
 
C
z ( z − 4)
2
= C ( z − 0)2 −1 3 X

A −1 B
2i  d  1  
=    ( Using Cauchy's integral formula )
1!  dz  z − 4  at z =0 Only z = 0 lies inside the curve
 −1  −i − j
= 2ix   = or
 ( z − 4 )2  8 8
 at z =0

55. A CMOS Schmitt-trigger inverter has a low output level of


0V and a high output level of 5V.
It has input thresholds of 1.6V and 2.4V.
10 k

47 nF

© All rights reserved by Thinkcell Learning Solutions Pvt. Ltd. No part of this booklet may be reproduced or utilized in any form without the written permission.
|EE-2021|

The input capacitance and output resistance of the Schmitt-trigger are negligible. The frequency of the
oscillator shown is _________Hz.

(Round off to 2 decimal places).


Key: (3166.20)
Sol:
V0
(volt)

0 t
T1 T2

1
T = T1 + T2 = 1, f =
T
( )
For T1 : VC 0− = 2.4V, VC ( ) = 0V
−t

(
VC ( t ) = VC (  ) + VC ( 0− ) − VC (  ) e RC )
At t = T1 , VC = 1.6V
− T1
1.6V = 0 + ( 2.4 − 0 ) e RC
− T1
1.6
e RC =
2.4
−T1  1.6   2.4 
= n  = − n 
RC  2.4   1.6 
 T1 = RC n (1.5 ) = 10 103  47 10−9  n (1.5 ) = 1.9 10−4 sec

( )
For T2 : VC 0− = 1.6V, VC ( ) = 5V

( )
t
VC ( t ) = VC (  ) + VC ( 0− ) − VC (  ) e

RC

At t = T2 ; VC = 2.4V

© All rights reserved by Thinkcell Learning Solutions Pvt. Ltd. No part of this booklet may be reproduced or utilized in any form without the written permission.
|EE-2021|
− T2
2.4V = 5 + (1.6 − 5 ) e RC
− T2
2.4 − 5 = −3.4e RC
− T2
−2.6 = −3.4e RC
− T2
2.6
e RC =
3.4
−T2  2.6   3.4 
= n  = − n 
RC  3.4   2.6 
T2 = RC n (1.307 )
= 10  103  47  10 −9  n (1.307 )
= 1.258  10 −4 sec
T = T1 + T2 = (1.9 + 1.258 ) 10−4 sec = 3.15110−4 sec

1 1
f= = Hz = 3166.201 Hz
T 3.15110−4
f = 3166.20Hz

© All rights reserved by Thinkcell Learning Solutions Pvt. Ltd. No part of this booklet may be reproduced or utilized in any form without the written permission.

You might also like